Smoking - GMAT Prep

This topic has expert replies
Senior | Next Rank: 100 Posts
Posts: 48
Joined: Wed Feb 26, 2014 10:53 pm

Smoking - GMAT Prep

by girishj » Sat Apr 30, 2016 8:34 pm
Why is B wrong and E correct?
Image

User avatar
Legendary Member
Posts: 2131
Joined: Mon Feb 03, 2014 9:26 am
Location: https://martymurraycoaching.com/
Thanked: 955 times
Followed by:140 members
GMAT Score:800

by MartyMurray » Sat Apr 30, 2016 10:12 pm
girishj wrote:Since smoking-related illnesses are a serious health problem in Country X, and since addiction to nicotine prevents many people from quitting smoking, the government of Country X plans to reduce the maximum allowable quantity of nicotine per cigarette by half over the next five years. However, reducing the quantity of nicotine per cigarette will probably cause people addicted to nicotine to smoke more cigarettes. Therefore implementing this plan is unlikely to reduce the incidence of smoking related illnesses.

Which of the following, if true, most strongly supports the argument about the consequences of implementing the Country X government's plan.

(A) Over half of the nonsmoking adults in Country X have smoked cigarettes in the past.

(B) If the Country X government's plan is implemented, the brands of cigarettes sold in Country X will differ less from each other than they do now in terms of their nicotine content.

(C) Inexpensive, smoke-free sources of nicotine, such as nicotine gum and nicotine skin patches, have recently become available in Country X.

(D) Many smokers in Country X already spend a large proportion of their disposable income on cigarettes.

(E) The main cause of smoking-related illnesses is not nicotine but tar in cigarette smoke.
Premise: Reducing the nicotine per cigarette will cause people to smoke more cigarettes.

Conclusion: Therefore the implementation of the plan will not reduce the incidence of smoking related health issues.

(A) This is irrelevant to an argument about the effects on people who smoke currently of a plan to reduce nicotine per cigarette.

(B) This sounds interesting but actually indicates little to nothing relevant to the argument. What it indicates is that currently some brands of cigarettes sold in Country X have higher nicotine levels than others do. However we don't know which brands are popular or anything much else that combined with what this answer choice says either weakens or supports the argument.

We already have reason to believe that the government's plan will reduce the level of nicotine per cigarette in some brands. So really what this choice says merely confirms that belief by indicating that the brands with higher levels of nicotine will become more like the brands that currently have lower levels.

(C) This does not support the argument. If anything it weakens the argument, as if cigarettes come to contain less nicotine people may decide to use one of the available alternative nicotine sources instead of cigarettes.

(D) This weakens the argument in that if people are already spending a large proportion of their income on cigarettes, they may be resistant to increasing their use of cigarettes.

(E) This supports the conclusion that the incidence of smoking-related illnesses will not be reduced by the implementation of the plan, in that if it is correct that people will smoke more cigarettes if there is less nicotine per cigarette, then given the information in this answer choice they will be taking in more tar, which is what causes smoking-related health problems.

The correct answer is E.
Marty Murray
Perfect Scoring Tutor With Over a Decade of Experience
MartyMurrayCoaching.com
Contact me at [email protected] for a free consultation.

Senior | Next Rank: 100 Posts
Posts: 48
Joined: Wed Feb 26, 2014 10:53 pm

by girishj » Sun May 01, 2016 3:26 am
Can you pls provide some more clarity or details for choice B. I didn't quite understand your reply.

User avatar
Legendary Member
Posts: 2131
Joined: Mon Feb 03, 2014 9:26 am
Location: https://martymurraycoaching.com/
Thanked: 955 times
Followed by:140 members
GMAT Score:800

by MartyMurray » Sun May 01, 2016 4:50 am
girishj wrote:Can you pls provide some more clarity or details for choice B. I didn't quite understand your reply.
The argument has two main aspects, a premise, that reducing nicotine levels will cause people to smoke more cigarettes, and a conclusion, that therefore the incidence of smoking-related illnesses will not be reduced by implementing the plan. So something that supports the argument will support the validity of the premise or more clearly connect the premise to the conclusion. In other words, currently even if the premise is valid and people will smoke more if the nicotine per cigarette is reduced, without making assumptions, for all we know their health will somehow be better. Maybe they will inhale somewhat less nicotine. Maybe smoking more cigarettes with less nicotine each is better than smoking fewer cigarettes with more nicotine each.

B does not do either of the things that I mentioned.

It does not confirm the validity of the premise that they will smoke more.

It does not do anything to connect the premise to the conclusion.

All that can be inferred from B is that currently some brands of cigarettes contain more nicotine than others do, and after the plan is implemented the levels of nicotine in the ones that have more will be reduced so that the levels in the ones that have more and the levels in the ones that have less will be more similar.

We already know that the plan is to reduce quantity of nicotine per cigarette. So given that we have no reason to believe that the differences in levels of nicotine between different brands of cigarettes are somehow pertinent to the argument, B adds basically nothing.

If you still feel that B somehow supports the argument, let me know why and I will directly address your thinking.
Marty Murray
Perfect Scoring Tutor With Over a Decade of Experience
MartyMurrayCoaching.com
Contact me at [email protected] for a free consultation.

Senior | Next Rank: 100 Posts
Posts: 39
Joined: Sat Jan 12, 2019 1:08 am

by meenakshimiyer » Sat Jan 12, 2019 8:39 pm
Conclusion: People who are already addicted to nicotine will start smoking more cigarettes to make up for the reduction of nicotine in a cigarette.
Statement E tells us that the Government is targeting the wrong substance. Even if people won't increase their cigarette consumption, they will still consume the same amount of tar, so there won't be a reduction in illnesses. Hence, E is the answer.